[CR][post office] HSPA posts

This topic has expert replies
User avatar
Legendary Member
Posts: 1101
Joined: Fri Jan 28, 2011 7:26 am
Thanked: 47 times
Followed by:13 members
GMAT Score:640

[CR][post office] HSPA posts

by HSPA » Wed May 11, 2011 3:14 am
I was stumped...LSAT

Brownlea's post office must be replaced with a larger one. The present one cannot
be expanded. Land near the present location in the center of town is more
expensive than land on the outskirts of town. Since the cost of acquiring a site is a
significant part of the total construction cost, the post office clearly could be built
more cheaply on the outskirts of town.

Which one of the following, if true, most seriously undermines the argument's
stated conclusion?

(A) The new post office will have to be built in accordance with a demanding
new citywide building code.

(B) If the new post office is built on the outskirts of town, it will require a parking
lot, but if sited near the present post office it will not.

(C) If the new post office is built on the outskirts of town, current city bus routes
will have to be expanded to provide access.

(D) If the new post office is built on the outskirts of town, residents will make
decreased use of post office boxes, with the result that mail carriers will have
to deliver more mail to homes.

(E) If the new post office is built near the center of town, disruptions to city
traffic would have to be minimized by taking such steps as doing some
construction work in stages at night and on weekends.
First take: 640 (50M, 27V) - RC needs 300% improvement
Second take: coming soon..
Regards,
HSPA.

User avatar
Master | Next Rank: 500 Posts
Posts: 234
Joined: Tue Feb 22, 2011 5:02 am
Thanked: 5 times
Followed by:3 members

by champmag » Wed May 11, 2011 4:30 am
IMO: D

Can u plz post the OA.

User avatar
Legendary Member
Posts: 1101
Joined: Fri Jan 28, 2011 7:26 am
Thanked: 47 times
Followed by:13 members
GMAT Score:640

by HSPA » Wed May 11, 2011 4:48 am
champmag wrote:IMO: D

Can u plz post the OA.
I too fell for the same trap.. try again.. OA will be provided only after few discussions
First take: 640 (50M, 27V) - RC needs 300% improvement
Second take: coming soon..
Regards,
HSPA.

Master | Next Rank: 500 Posts
Posts: 165
Joined: Wed Aug 04, 2010 9:44 pm
Thanked: 8 times

by sandy217 » Wed May 11, 2011 6:56 am
I came down to B and C , because argument has to be weakened by introducing cost factor.
Initially i chose C, but i think it should be B because building parking lot definitely incurs additional cost.

User avatar
Senior | Next Rank: 100 Posts
Posts: 42
Joined: Mon Oct 18, 2010 11:39 pm

by novel » Wed May 11, 2011 8:02 am
I came down to b and d
a,c and e are relevant

IMO b -as it mentions that large are will be required.This will increase the cot.IN that case constructing post office on out skirts may not economical in comparison to extension of existing one.

User avatar
Senior | Next Rank: 100 Posts
Posts: 37
Joined: Tue Jan 18, 2011 2:50 am
Thanked: 2 times

by abidshariff » Wed May 11, 2011 8:02 am
IMO B. because all of the other choices deal with the problems that are out of scope. And only choice B deals with the cost problem, the problem clearly discussed in the question(' cheaply in the last sentence')

User avatar
Legendary Member
Posts: 1101
Joined: Fri Jan 28, 2011 7:26 am
Thanked: 47 times
Followed by:13 members
GMAT Score:640

by HSPA » Wed May 11, 2011 8:32 am
Yes OA is B
First take: 640 (50M, 27V) - RC needs 300% improvement
Second take: coming soon..
Regards,
HSPA.